[obm-l] Re: [obm-l] Aritmética

2015-10-24 Por tôpico Willy George Amaral Petrenko
Seja k um divisor impar de m e n. Observe que an + bm = akn' + bkm' = (an'
+ bm') * (a(k-1)n' - a(k-2)n'bm' +  - an b(k-2)m'+ b(k-1)m').

Bom, a partir daí vc preenche os detalhes. Só acrescento que há uma
exceção, quando a=b=1, n e m podem ter valores arbitrários e a soma dá
sempre 2 que é primo.

att

2015-10-25 0:52 GMT-02:00 marcone augusto araújo borges <
marconeborge...@hotmail.com>:

> Alguém poderia resolver?
>
> Sejam a, b, n, m inteiros positivos e suponha que a^n + b^m seja
> um número primo.Mostre que (n,m) = 1 ou (n,m) = 2^r, para algum
> r inteiro positivo.
> Desde já agradeço.
>
>
>
> --
> Esta mensagem foi verificada pelo sistema de antivírus e
> acredita-se estar livre de perigo.
>

-- 
Esta mensagem foi verificada pelo sistema de antiv�rus e
 acredita-se estar livre de perigo.



[obm-l] Re: [obm-l] Números complexos

2014-09-08 Por tôpico Willy George Amaral Petrenko
Vc quer uma dica ou a solução?

Dica: Lembre que pela forma trigonométrica, o seno de um ângulo tem a ver
com a parte imaginária. Observe que se vc calcular a parte imaginária na
igualdade acima, o 1 morre.

Se quiser a solução responde.

2014-09-08 8:05 GMT-03:00 Vanderlei Nemitz vanderma...@gmail.com:

 Pessoal, estou precisando de uma grande ajuda em um problema do livro do
 Manfredo. Pede para mostrar a lei dos senos utilizando números complexos:

 *No triângulo ABC onde a, b e c são os lados opostos aos ângulos A, B e C,
 respectivamente, demonstre que *

 *a/senA = b/senB = c/senC ( Lei dos senos)*

 *Sugestão: Considere complexos z1, z2 e z3 cujas imagens são os vértices
 do triângulo e use a identidade (z1 – z2)/(z3 – z1) + (z2 – z3)/(z3 – z1) +
 1 = 0.*

 Se alguém puder me dar uma dica, pois não consegui perceber como e onde
 utilizar a identidade sugerida.

 Obrigado,

 Vanderlei

 --
 Esta mensagem foi verificada pelo sistema de antivírus e
 acredita-se estar livre de perigo.

-- 
Esta mensagem foi verificada pelo sistema de antiv�rus e
 acredita-se estar livre de perigo.



[obm-l] Re: [obm-l] Re: [obm-l] Re: [obm-l] Números complexos

2014-09-08 Por tôpico Willy George Amaral Petrenko
A = z1; B = z2; C = z3

(z1-z2) é o vetor correspondente ao lado c. (z1-z2)/(z1-z3) é um complexo
que tem argumento igual ao ângulo Â. Então pela igualdade:

(z1-z2)/(z1-z3) + (z2-z3)/(z1-z3) + 1 = Im{(z1-z2)/(z1-z3)} + Im{(z2-z3)/(z
1-z3)} = 0 = |(z1-z2)/(z1-z3)| * sen  =  |(z2-z3)/(z1-z3)| * sen C = |(z1
-z2)| * sen  =  |(z2-z3)| * sen C = c senA = a senC = a/senA = c/senC.
cqd

2014-09-08 12:31 GMT-03:00 Vanderlei Nemitz vanderma...@gmail.com:

 Willy, se não for incomodar, poste a solução. Preciso desse resultado para
 prosseguir.

 Muito obrigado pela ajuda!

 Vanderlei

 Em 8 de setembro de 2014 12:24, Willy George Amaral Petrenko 
 wgapetre...@gmail.com escreveu:

 Vc quer uma dica ou a solução?

 Dica: Lembre que pela forma trigonométrica, o seno de um ângulo tem a ver
 com a parte imaginária. Observe que se vc calcular a parte imaginária na
 igualdade acima, o 1 morre.

 Se quiser a solução responde.

 2014-09-08 8:05 GMT-03:00 Vanderlei Nemitz vanderma...@gmail.com:

 Pessoal, estou precisando de uma grande ajuda em um problema do livro do
 Manfredo. Pede para mostrar a lei dos senos utilizando números complexos:

 *No triângulo ABC onde a, b e c são os lados opostos aos ângulos A, B e
 C, respectivamente, demonstre que *

 *a/senA = b/senB = c/senC ( Lei dos senos)*

 *Sugestão: Considere complexos z1, z2 e z3 cujas imagens são os vértices
 do triângulo e use a identidade (z1 – z2)/(z3 – z1) + (z2 – z3)/(z3 – z1) +
 1 = 0.*

 Se alguém puder me dar uma dica, pois não consegui perceber como e onde
 utilizar a identidade sugerida.

 Obrigado,

 Vanderlei

 --
 Esta mensagem foi verificada pelo sistema de antivírus e
 acredita-se estar livre de perigo.



 --
 Esta mensagem foi verificada pelo sistema de antivírus e
 acredita-se estar livre de perigo.



 --
 Esta mensagem foi verificada pelo sistema de antivírus e
 acredita-se estar livre de perigo.

-- 
Esta mensagem foi verificada pelo sistema de antiv�rus e
 acredita-se estar livre de perigo.



[obm-l] Re: [obm-l] Re: [obm-l] Re: [obm-l] Re: [obm-l] Re: [obm-l] Números complexos

2014-09-08 Por tôpico Willy George Amaral Petrenko
Ah sim, eu fui um pouco descuidado com o sinal _ o que quer dizer que eu
errei :( mas a ideia está certa:)

Im{(z1-z2)/(z1-z3)} + Im{(z2-z3)/(z1-z3)} = 0 = Im{(z1-z2)/(z1-z3)} = Im{(z
3-z2)/(z1-z3)}

Aqui só se pode afirmar que Im{(z1-z2)/(z1-z3)} = |(z1-z2)/(z1-z3)| * sen
Â, dependendo da orientação do triângulo (ou seja, dependendo se o complexo
z1-z2 tem argumento maior do que o complexo z1-z3). Caso contrário seria
..sen - Â. Mas aí vc repara que independente da orientação, ambos Im{(z1
-z2)/(z1-z3)} e Im{(z3-z2)/(z1-z3)} tem o mesmo sinal. Daí, tendo em vista
que sen (- Â) = - sen Â, segue o raciocínio normalmente.

2014-09-08 22:15 GMT-03:00 Vanderlei Nemitz vanderma...@gmail.com:

 Valeu, Willy! Só não ficou muito clara a seguinte passagem:

  Im{(z1-z2)/(z1-z3)} + Im{(z2-z3)/(z1-z3)} = 0 = |(z1-z2)/(z1-z3)| * sen
 Â  =  |(z2-z3)/(z1-z3)| * sen C

 Como que a soma nula transformou-se em uma igualdade?

 Obrigado!


 Em 8 de setembro de 2014 13:07, Willy George Amaral Petrenko 
 wgapetre...@gmail.com escreveu:

 A = z1; B = z2; C = z3

 (z1-z2) é o vetor correspondente ao lado c. (z1-z2)/(z1-z3) é um
 complexo que tem argumento igual ao ângulo Â. Então pela igualdade:

 (z1-z2)/(z1-z3) + (z2-z3)/(z1-z3) + 1 = Im{(z1-z2)/(z1-z3)} + Im{(z2-z3
 )/(z1-z3)} = 0 = |(z1-z2)/(z1-z3)| * sen  =  |(z2-z3)/(z1-z3)| * sen C
 = |(z1-z2)| * sen  =  |(z2-z3)| * sen C = c senA = a senC = a/senA =
 c/senC. cqd

 2014-09-08 12:31 GMT-03:00 Vanderlei Nemitz vanderma...@gmail.com:

 Willy, se não for incomodar, poste a solução. Preciso desse resultado
 para prosseguir.

 Muito obrigado pela ajuda!

 Vanderlei

 Em 8 de setembro de 2014 12:24, Willy George Amaral Petrenko 
 wgapetre...@gmail.com escreveu:

 Vc quer uma dica ou a solução?

 Dica: Lembre que pela forma trigonométrica, o seno de um ângulo tem a
 ver com a parte imaginária. Observe que se vc calcular a parte imaginária
 na igualdade acima, o 1 morre.

 Se quiser a solução responde.

 2014-09-08 8:05 GMT-03:00 Vanderlei Nemitz vanderma...@gmail.com:

 Pessoal, estou precisando de uma grande ajuda em um problema do livro
 do Manfredo. Pede para mostrar a lei dos senos utilizando números 
 complexos:

 *No triângulo ABC onde a, b e c são os lados opostos aos ângulos A, B
 e C, respectivamente, demonstre que *

 *a/senA = b/senB = c/senC ( Lei dos senos)*

 *Sugestão: Considere complexos z1, z2 e z3 cujas imagens são os
 vértices do triângulo e use a identidade (z1 – z2)/(z3 – z1) + (z2 –
 z3)/(z3 – z1) + 1 = 0.*

 Se alguém puder me dar uma dica, pois não consegui perceber como e
 onde utilizar a identidade sugerida.

 Obrigado,

 Vanderlei

 --
 Esta mensagem foi verificada pelo sistema de antivírus e
 acredita-se estar livre de perigo.



 --
 Esta mensagem foi verificada pelo sistema de antivírus e
 acredita-se estar livre de perigo.



 --
 Esta mensagem foi verificada pelo sistema de antivírus e
 acredita-se estar livre de perigo.



 --
 Esta mensagem foi verificada pelo sistema de antivírus e
 acredita-se estar livre de perigo.



 --
 Esta mensagem foi verificada pelo sistema de antivírus e
 acredita-se estar livre de perigo.

-- 
Esta mensagem foi verificada pelo sistema de antiv�rus e
 acredita-se estar livre de perigo.



[obm-l] Re: [obm-l] Re: [obm-l] Raízes irracionais

2014-08-07 Por tôpico Willy George Amaral Petrenko
Bem, o Bernardo já corrigiu o enunciado, então vou partir daí.

Vc sabe álgebra avançada? Anéis, corpos, ideais, domínios euclidianos,
anéis quociente, anéis de polinômios? Seria o ideal (pun intended) para
entender a demonstração. Mas talvez dê para pegar a ideia sem isso, vou
tentar ser didático.

Um negócio sobre o conjunto dos polinômios de coeficientes racionais
(chamado Q[x]), é que ele é um domínio euclidiano. Dentre outras
propriedades, isso significa que vc pode fazer uma divisão de polinômios,
chamada divisão euclidiana:

Sejam f e g ∈ Q[x]. Então existem únicos q e r ∈ Q[x], com grau (r)  grau
(g) tais que f = g*q + r

Isso não é difícil de provar, dá pra fazer por indução no grau.

Considere um polinômio do tipo g(x) = x2 - m, que não possui raiz racional
(ou seja, as raiz dele são √m e -√m, ambas irracionais). Seja p(x), um
polinômio racional com raiz a+b√m. Definimos f(x) = p(bx+a). É fácil ver
que f tem √m como raiz. Além disso, ele continua sendo um polinômio
racional. Gostaríamos de provar que f também possui -√m como raíz. Façamos
a divisão de f por g:

f(x) = (x2 - m)*q(x) + r(x), com grau (r)  grau (x2 - m) = 2. Ou seja, r é
de grau no máximo 1 e portanto é da forma r(x) = cx + d. Mas f(√m) = 0:

f(√m) = (√m2 - m)*q(√m) + r(√m) == 0 = r(√m) == c√m + d = 0. Mas lembre
que r é um polinômio racional também, logo c = d = 0, pois caso contrário
teríamos -d/c = √m, absurdo pois √m não é racional.

Tudo isso significa que r(x) é identicamente nulo, ou seja, que f é
divisível por x2 - m. Logo ele também possui -√m como raíz. Aplicando isso
na definição de f, descobrimos que o polinômio original tem a-b√m como
raiz, cqd.


Para a parte 2, temos que considerar o corpo Q2 = {a√m + b, a,b ∈ Q}. Ele
também é euclidiano. Considere os polinômios em Q2 = Q2[x]. Esse conjunto é
uma extensão de Q[x] - de fato, todo polinômio com coeficientes racionais
tem coeficientes da forma a√m + b com a = 0. A demonstração segue análoga à
da parte 1, só que agora g(x) = x2 - n, que não possui raízes em Q2. Além
disso, p(x) tem raiz a + b√m + c√n, e vc vai definir o f ∈ Q2 de forma que
ele tenha √n como raíz, ou seja f(x) = p(a + b√m + cx).

Obviamente o resultado pode ser facilmente extendido por indução para n
raízes independentes


Abç

Willy


2014-08-07 8:47 GMT-03:00 Bernardo Freitas Paulo da Costa 
bernardo...@gmail.com:

 2014-08-07 7:21 GMT-03:00 Pedro Chaves brped...@hotmail.com:
  Prezados Colegas,
 
  Gostaria de saber se alguém conhece uma demonstração do teorema abaixo.
  Um abraço do Pedro Chaves!
  ___
 
  Teorema das raízes irracionais:
 
 
  Seja P(x) um polinômio não identicamente nulo e de coeficientes
 racionais, e sejam a, m e n números racionais
  — m e n são positivos e m^(1/2) e n^(1/2) são números irracionais. Sejam
 M = m^(1/2) e N = n^(1/2). Pode-se então afirmar:
 
  1) Se a + M é raiz de P(x), então a - M também o é (e com a mesma
 multiplicidade).
 
  2) Se M + N é raiz de P(x), então M - N, -M + N e -M-N também o são (e
 todas as quatro raízes têm a mesma multiplicidade).

 Essa segunda é falsa. Seja M = raiz(2) e N = 2*raiz(2), de forma que m
 = 2 e n = 8. Seja agora P(x) um polinômio com raiz M + N = 3*raiz(2).
 Basta tomar P(x) = x^2 - 18. Para este polinômio, -(M+N) também é
 raíz, mas nem M-N nem -(M-N) são.

 Para ser verdade, você precisa que M e N sejam racionalmente
 independentes, o que é (quase) o que você quer mostrar no teorema...

 Abraços,
 --
 Bernardo Freitas Paulo da Costa

 --
 Esta mensagem foi verificada pelo sistema de antivírus e
  acredita-se estar livre de perigo.


 =
 Instru�ões para entrar na lista, sair da lista e usar a lista em
 http://www.mat.puc-rio.br/~obmlistas/obm-l.html
 =


-- 
Esta mensagem foi verificada pelo sistema de antiv�rus e
 acredita-se estar livre de perigo.



[obm-l] Re: [obm-l] Questão da 3ª fase nível 1 da OBM 2013

2014-04-23 Por tôpico Willy George Amaral Petrenko
vc quer uma ajuda ou uma solução?

Uma ajuda:

a) Observe que 22 não tem muitos divisores próprios, apenas 1,2 e 11.
Mostre que se 11 fizer parte do quadrado, então algum outro múltiplo de 11
além do 22 também estará (ou seja, não existe quadrado onde os únicos
múltiplos de 11 sejam 11 e 22). Assim se 22 for o maior, 11 não pode estar
no quadrado.

Portanto 22 deverá estar ao lado de 1 e 2, logo deve estar no canto:

22  1  x
2   x   x
x   x   x

Agora eu fiz por tentativa e erro.

b) Generalize a observação que eu fiz para um primo qualquer (ou seja se p
está no quadrado, então outro múltiplo de p que não é o 2p também estará).
Tente achar um quadrado cujo maior número seja o menor que conseguir e use
a observação para mostrar que não existe nenhum menor.

Se não conseguir fazer eu posso mandar a solução.


2014-04-23 18:37 GMT-03:00 Érica G. Pongelupe Giacoia profer...@ig.com.br:



 Prezados,
 Gostaria da ajuda de vcs para resolver a seguinte questão que caiu na 3ª
 fase (nível 1) da OBM do ano passado.

 Desde já obrigada a todos.

 Érica G. P. Giacoia

 (OBM 2013) Desejamos preencher um tabuleiro 3x3 com 9 inteiros positivos
 distintos sendo que números a e b que têm um lado em comum devem ser tais
 que a é divisível por b ou b é divisível por a.

 Vejamos uma configuração que satisfaz as condições do problema. Observe
 que o maior número que aparece no tabuleiro é o 25.

 8 2   10

 420   5

 12   1   25

 a) Apresente uma maneira de preencher um tabuleiro de modo que o maior
 número que aparece é  22.

 b) Qual é o menor inteiro positivo que pode ser o maior número que aparece
 no tabuleiro?



 --
 Esta mensagem foi verificada pelo sistema de antivírus e
 acredita-se estar livre de perigo.


-- 
Esta mensagem foi verificada pelo sistema de antiv�rus e
 acredita-se estar livre de perigo.



[obm-l] Re: [obm-l] Re: [obm-l] Álgebra(não tá saindo)

2013-09-14 Por tôpico Willy George Amaral Petrenko
Ou resolva a equação em *N*:

(10*x+6)*4 = 6*10n + x = 39*x + 24 = 6*10n = 13*x = 2*10n - 8 = 10n = 4 mod
13 = n = 5 + 12k. Logo o menor n é 5 e o menor número é (2*105 - 8)/13 =
15384  Obviamente vc adiciona o 6 depois: 153846


2013/9/14 Ralph Teixeira ralp...@gmail.com

 Escreva a multiplicacao que nem a gente fazia lah na 4a serie:
 _6
   x4
 6_

 Agora vah fazendo a multiplicacao. 6x4=24, entao poe o 4, vai 2.
 Mas, se eh 4 ali embaixo, eh 4 do lado esquero do 6. Entao fica algo assim:
 46
x4
 64
 Agora 4x4=16, mais 2, dah 18. Entao poe o 8 no resultado E TAMBEM DO LADO
 DO 4 NA PRIMEIRA LINHA (e vai 1).
 ___846
x4
 6___84
 4x8=32, +1=33. O proximo eh 3. Continue assim achando os digitos da
 direita para a esquerda: 5, 1... E entao o proximo eh 6, que PODE ser
 aquele 6 inicial!

 Assim, o menor numero inteiro n eh 153846.

 Abraco,
 Ralph



 2013/9/14 marcone augusto araújo borges marconeborge...@hotmail.com

 Encontre o menor inteiro positivo n que possui as seguintes propriedades:
 I. Em sua representação tem o 6 como último dígito
 II.Se o último dígito(6) é apagado  e colocado na frente dos dígitos
 restantes,o número resultante
 é quatro vezes maior que o número original n

 --
 Esta mensagem foi verificada pelo sistema de antivírus e
 acredita-se estar livre de perigo.



 --
 Esta mensagem foi verificada pelo sistema de antivírus e
 acredita-se estar livre de perigo.


-- 
Esta mensagem foi verificada pelo sistema de antivírus e
 acredita-se estar livre de perigo.



[obm-l] Re: [obm-l] Re: [obm-l] Soma de radicais irracionais é irracional

2013-09-09 Por tôpico Willy George Amaral Petrenko
O caso geral é meio complicado. Mas vou dar uma ideia de como se prova que √
2 + 3√3 é irracional.

Primeiro introduzimos o conjunto Q[√2], que é o menor corpo que contem
tanto Q quanto √2. Ele é formado pelos caras da forma a + b√2, onde a,b ∈
Q. Suponha que √2 + 3√3 ∈ Q[√2]. Então existem a,b tal que √2 + 3√3 = a + b√
2. Mas então temos que 3√3 ∈ Q[√2] também. Daí temos a,b ∈ Q com 3 = (a + b√
2)3⇒ 3 = a3. Prossiga de maneira similar a prova tradicional de que 3√3 é
irracional. Mas nesse caso vc prova √2 +3√3 não pertence a Q[√2]. Mas como
Q ⊂ Q[√2], temos a resposta.

Bem, alguns passos são não triviais, mas essa é a ideia.




2013/9/7 terence thirteen peterdirich...@gmail.com

 Complicadinho...

 Primeiro, dá para supor que a1/m e b1/n estão reduzidos.

 Acho que a forma seria obter um polinômio que tenha esta soma como raiz, e
 provar que nenhum racional pode ser raiz deste polinômio.

 Por exemplo,

 21/2+31/3=x
 81/6+91/6=x

 Assim, podemos de alguma forma supor que x é raiz de um polinômio de grau
 6(acho que alguma coisa relacionada a Álgebra Linear pode provar isto).

 De qualquer forma, calculamos xn, com n de 1 até 6, e tentamos obter
 alguma combinação linear entre as alternativas, para daí obter o polinômio
 de grau 6. MAS como demonstrar que nenhum racional pode ser raiz deste
 polinômio?






 Em 26 de agosto de 2013 19:19, Ennius Lima enn...@bol.com.br escreveu:

 Caros Colegas,

 Sendo a, b, m e n inteiros positivos tais que a1/m e b1/n são
 irracionais, como podemos provar que a soma a1/m + b1/n também é
 irracional?

 Abraços do Ennius!
 __
 Â

 --
 Esta mensagem foi verificada pelo sistema de antivírus e
  acredita-se estar livre de perigo.

 =
 Instruções para entrar na lista, sair da lista e usar a lista em
 http://www.mat.puc-rio.br/~obmlistas/obm-l.html
 =




 --
 /**/
 神が祝福

 Torres

 --
 Esta mensagem foi verificada pelo sistema de antivírus e
 acredita-se estar livre de perigo.

-- 
Esta mensagem foi verificada pelo sistema de antiv�rus e
 acredita-se estar livre de perigo.



Re: [obm-l] Sair da lista.

2013-05-03 Por tôpico Willy George Amaral Petrenko
Isso é que nem vida após a morte: Quem tá dentro não sabe, quem tá fora não
volta pra dizer!


2013/5/3 felipe baltor felipebal...@hotmail.com

 Pessoal, eu tentei sair da lista com as instruções normais mas não tô
 conseguindo. Será que alguém pode me ajudar?




Re: [obm-l] Problema 4 Cone Sul 1996

2013-04-30 Por tôpico Willy George Amaral Petrenko
Eu acho que não entendi o enunciado.
A cada passo apenas um número é mudado, ou não? E é mudado pela média
aritmética dele com alguns outros?


2013/4/30 Carlos Yuzo Shine cysh...@yahoo.com

 O erro foi supor que na situação anterior os números na sequência ficariam
 a,b,b,b,...,b.

 Poderia muito bem ser, digamos, 997/998, 1, 1995/1996, 1995/1996, ...,
 1995/1996.

 Se você ainda quer pensar no problema, pare de ler aqui. Caso contrário,
 continue.

 O que você pode fazer para resolver o problema é fazer a média dos
 primeiros 998 números, obtendo 998 números iguais a 997/998 e depois fazer
 pares com 997/998 e 1 (fazendo a operação mais 998 vezes). Note que esse
 argumento funciona para qualquer número composto no lugar do 1996.

 E no caso em que trocamos 1996 por um primo p (um 0 e p-1 uns)? Aí não dá,
 porque no final o denominador tem que p (todo mundo teria que ser igual a
 (p-1)/p, já que a soma de todos nunca muda), e isso obrigaria a gente a, em
 algum momento, dividir tudo por p, o que não é possível.

 Mas e se a soma dos p números é múltiplo de p? Mais uma boa pergunta, não?

 []'s
 Shine

 
 From: EPVN barz...@dglnet.com.br
 To: obm-l@mat.puc-rio.br
 Sent: Monday, April 22, 2013 11:57 AM
 Subject: [obm-l] Problema 4 Cone Sul 1996




 O enunciado é:

 A seqüência 0, 1, 1, 1,
 ... , 1 contém 1996 números, sendo o primeiro zero e todos os demais um.
 Se escolhem
 dois ou mais números da seqüência (mas não todos) e se sustitui um deles
 pela
 média aritmética dos números escolhidos, obtendo-se assim uma nova
 seqüência de
 1996 números.
 Provar que, com a
 repetição desta operação, é possível obter uma seqüência na qual os 1996
 números são iguais.

 NOTA: Não é necessário
 escolher a mesma quantidade de números em cada operação.

 Um colega apresentou a seguinte argumentação:

 Se essa operação levasse a uma seqüência com todos os números
 idênticos então no penúltimo estágio teríamos algo assim:

 a,b,b,b,..,b , com um único número diferente que
 deve ser tornado igual aos demais com mais um passo. Bem, se tomarmos p
 números
 b e mais o número a,
 obteremos o número (a + pb)/ (p + 1 ), igualando a b teríamos
 a=b.
 Parece que isso prova que esse penúltimo estágio nunca é
 atingido e, portanto, o último também não.

 Se algum colega puder nos ajudar a esclarecer a situação
 ficamos muito gratos.

 Um abraço.

 Osmundo Bragança.

 =
 Instruções para entrar na lista, sair da lista e usar a lista em
 http://www.mat.puc-rio.br/~obmlistas/obm-l.html
 =



[obm-l] Re: [obm-l] Equipes Mínimas de campeonatos

2013-04-07 Por tôpico Willy George Amaral Petrenko
Ou eu não entendi o enunciado, ou ele está errado.

Para n fixado, seja o conjunto de equipes 1,2,3,...,n, onde n ganha de todo
mundo, n-1 ganha de todos exceto o n, etc. Ou seja, o resultado da partida
i,j é max(i,j).
Esse torneio satisfaz o enunciado, mas não satisfaz i nem ii.


2013/4/6 Jeferson Almir jefersonram...@gmail.com

 Esse que essa tematica é muito recorrente aqui então queria uma ajuda
 nessa questão

 Em um torneio cada equipe joga exatamente uma única vez com as equipes
 restantes. No torneio participam ao menos n equipes. Se para cada grupo de
 n equipes participantes existe uma equipe que perdeu para todas equipes de
 seu grupo. Prove que:

 i.Para cada grupo A de n-1 equipes existe um grupo B de n+1 equipes
 tais que cada equipe de A já ganhou de cada equipe de B.

 ii.No torneio há pelo menos equipes (n+2)2ˆ(n-1) -1 participantes.

 --
 Esta mensagem foi verificada pelo sistema de antivírus e
 acredita-se estar livre de perigo.

-- 
Esta mensagem foi verificada pelo sistema de antiv�rus e
 acredita-se estar livre de perigo.



Re: [obm-l] Calcular o mdc (333...3, 333...3)

2012-12-04 Por tôpico Willy George Amaral Petrenko
O 1o numero é (10^100 - 1)/3, enquanto o 2o é (10^80 - 1)/3. Obviamente eu
posso ignorar esse 1/3 aí, e depois dividir a resposta que eu achar por 3.

Então quero calcular mdc entre (x^10 - 1) e (x^8 - 1), onde x=10^10.

Então eu percebo que x^2 - 1 divide ambos (se eu não percebesse, eu sempre
poderia fazer a divisão euclidiana deles). Dividindo tudo por x^2 - 1 fica:

p(x) = x^8 + x^6 + x^4 + x^2 + 1 e q(x) = x^6 + x^4 + x^2 + 1. Então eu
percebo que p(x) - x^2*q(x) = 1 (novamente se eu não percebesse eu faria a
divisão...). Então acabou pq se alguém divide p(x) e q(x) para algum x,
então também divide 1. Logo esses caras são primos entre si. Assim o mdc
original fica (x^2-1)/3 = 10^20 - 1



2012/12/4 Pedro Chaves brped...@hotmail.com

  Colegas da Lista,

 Como calcular o mdc (a, b) , sendo a = 333...3  (100 dígitos iguais a 3) e
 b = 333...3 (80 dígitos iguais a 3)?


 Abraços do pedro Chaves

 _-




Re: [obm-l] Produto na base 7

2012-10-31 Por tôpico Willy George Amaral Petrenko
Pelo que eu entendi ele escreveu esses números na base 7
(1,2,3,4,5,6,10...). Nesse caso os múltiplos de 7 são 10, 20, 30, 40, 50,
60, ou seja 6 zeros

2012/10/31 Marcelo Salhab Brogliato msbro...@gmail.com

 Olá, Ennius,

 Seja A = 1x2x3x...x66 = Sum{i=0..n} a_i 7^i.

 Como 7 é primo, temos que ver quantas vezes o fator 7 está aparecendo
 nesse produtório.
 Temos o fator 7 em: 7, 14, 21, 28, 35, 42, 49, 56, 63.
 No 49 ele aparece 2 vezes, logo, temos um total de 10 fatores 7.

 Portanto, temos a0 = a1 = a2 = a3 = ... = a9 = 0. Isso é, são 10 zeros. :)

 Abraços,
 Salhab


 2012/10/31 ennius enn...@bol.com.br

 Caros Colegas,


 Na base 7, em quantos zeros termina o produto 1 x 2 x 3 ... x 66? Â (Os
 fatores estão na base 7.)

 Abraços do Ennius


 __
 =
 Instruções para entrar na lista, sair da lista e usar a lista em
 http://www.mat.puc-rio.br/~obmlistas/obm-l.html
 =





Re: [obm-l] sair da lista

2012-10-18 Por tôpico Willy George Amaral Petrenko
A galera está mandando esses emails porque o link regular 
http://www.mat.puc-rio.br/~obmlistas/obm-l.html; está quebrado. Se alguém
tiver acesso a isso é bom tentar consertar.



2012/10/17 Rita Gomes rcggo...@terra.com.br

 Luis,


 O problema é que eu nao tenho interagido aqui e nao justifica ficar na
 lista se nao estou participando. Não estou tendo tempo para sequer ler os
 emails recebidos, quanto mais analisar as questões que chegam aqui.

 Sei que são bastante proveitosas e ja tirei muito proveito dessa lista com
 duvidas que tive, mas no momento esta apenas enchendo a minha caixa de
 correio.


 Rita Gomes



 *On Ter 16/10/12 20:42 , Luís Junior jrcarped...@gmail.com sent:
 *

 Ahh Rita, fica vai... vou me sentir sozinho e com saudades!

 2012/10/16 Rita Gomes rcggo...@terra.com.br


 Quero sair da lista






Re: [obm-l] JEITO CEARENSE!

2012-09-09 Por tôpico Willy George Amaral Petrenko
Na verdade p^2+(1-p)^2 =1/2 das possibilidades são descartadas.

2012/9/9 Jorge Luis Rodrigues e Silva Luis jorgelrs1...@hotmail.com

  Olá Pessoal!  Ainda com relação ao vício da moeda do Jeferson observem
 que metade das possibilidades são descartadas...Como bolar uma variante ao
 esquema tal que minimizasse essa perda? Será que o Neumann acharia uma
 saída?

 Numa faculdade há dois cursos e um rapaz e uma moça estão trocando idéias.
 O rapaz diz: Aqui eles discriminam contra os homens, a proporção de homens
 admitidos (dentre os candidatos) é menor do que a de mulheres. A moça
 responde: Não, eles discriminam contra as mulheres. Nos dois cursos a
 proporçào de mulheres admitidas (dentre as candidatas) é menor do que a de
 homens. É possivel que ambos tenham razào quanto aos fatos?  (Proposto
 pelo Nicolau!)


 A propósito! De quantas formas podemos colocar N rainhas em um tabuleiro N
 * N tal que nenhuma rainha possa enxergar outra?


 Abraços!



[obm-l] Re: [obm-l] Re: [obm-l] [obm-l] Re: [obm-l] problema da divisão

2012-08-16 Por tôpico Willy George Amaral Petrenko
O passo 6 está ambíguo. E se ambos B e C marcarem os mesmos pedaços como
ruins?

2012/8/15 Manoel R D'Oliveira Neto dol...@mac.com

 A solução publicada no livro How to Cut a Cake, de Ian Stewart, foi
 apresentada em 1944 pelo matemático polonês Hugo Steinhaus, quando esteve
 prisioneiro do exército alemão (no exemplo do livro não era uma herança e
 sim um bolo):

 1) A corta o bolo em 3 pedaços;
 2) B pode passar (se achar que ao menos 2 dos pedaços são justos) ou
 marcar dois pedaços como 'ruins';
 3) Se B passou, então C escolhe primeiro um pedaço, B escolhe a seguir um
 outro pedaço e A pega o pedaço que sobrou;
 4) Se B marcou dois pedaços como 'ruins', então C recebe as mesmas opções
 que B, sem ficar sabendo dos pedaços marcados por B;
 5) Se C passou, então os pedaços são escolhidos na seguinte ordem: B, C e
 A;
 6) Caso contrário, então tanto B quanto C marcaram dois pedaços como
 'ruins'. Deve haver pelo menos 1 pedaço que ambos considerem 'ruins'. Esse
 pedaço fica com A;
 7) Os outros dois pedaços são então empilhados. E, entre B e C, um corta e
 o outro escolhe a parte que achar melhor, ficando a parte restante para o
 que cortou a pilha em dois.

 Sds,
 Manoel DOliveira


 On 15/08/2012, at 15:10, Samuel Wainer wrote:

 É sim. Mas é que fiquei imaginando qual seria o método de estipular qual
 seria a ordem de quem iria pegar. Porque se o primeiro divide, mesmo que
 ele seja o último, quem seria o primeiro? E se ele tivesse combinado com
 esse primeiro?
 Talvez minha dúvida não faça sentido.

 --
 Date: Tue, 14 Aug 2012 16:03:07 -0700
 From: eduardowil...@yahoo.com.br
 Subject: [obm-l] Re: [obm-l] problema da divisão
 To: obm-l@mat.puc-rio.br

 Me parece que estipulando que aquele que divide é o ultimo a pegar sua
 parte, resolve.

 Ou não é este o espírito da questão?

 [ ]'s




[obm-l] Re: [obm-l] Triângulo

2012-04-01 Por tôpico Willy George Amaral Petrenko
Use a desigualdade triangular, que é condição necessária e suficiente para
existência de um triângulo com lados l1, l2, l3



2012/4/1 marcone augusto araújo borges marconeborge...@hotmail.com

  Em que condições as medidas dos lados de um triângulo estão em PG?


 Se for um triangulo retangulo,a razão da PG será q = raiz((1+raiz(5))/2) e
 o cosseno de um dos seus angulos agudos será 1/q.
 Se isso é verdade,restariam os casos dos triangulos acutangulos e dos
 obtusangulos.



[obm-l] Re: [obm-l] Re: [obm-l] Re: [obm-l] Re: [obm-l] Convergência de Matriz

2011-11-22 Por tôpico Willy George Amaral Petrenko
Repare que parte do problema é:
aij  0 para todo i,j

2011/11/22 Jaare Oregim jaare.ore...@gmail.com

 nao vale para a matriz 2x2
  0  1
  1  0

 se for o caso, é corolário do Perron–Frobenius para matrizes
 irredutiveis (que nao é o caso do exemplo acima)

 http://en.wikipedia.org/wiki/Perron%E2%80%93Frobenius_theorem#Perron.E2.80.93Frobenius_theorem_for_irreducible_matrices


 On Sun, Nov 20, 2011 at 7:52 AM, Artur Costa Steiner
 steinerar...@gmail.com wrote:
  E as colunas são iguais ao auto-vetor correspondente ao auto-valor 1 tal
 que
  a soma das componentes é 1.
 
  Artur
 
  Artur Costa Steiner
 
  Em 19/11/2011 00:10, Willy George Amaral Petrenko 
 wgapetre...@gmail.com
  escreveu:
 
  Esse problema é meio complicado, mas ele é um corolário desse teorema:
  http://en.wikipedia.org/wiki/Perron%E2%80%93Frobenius_theorem
 
 
  2011/11/17 Henrique Rennó henrique.re...@gmail.com
 
  Suponha a matriz [aij]mxm, onde cada aij representa a probabilidade de
 um
  evento i levar a um evento j e aij  0 para todo i,j. Calculando as
  potências dessa matriz, é possível provar que ela converge para uma
 matriz P
  em que todas as colunas são iguais?
  Vi essa propriedade no livro Álgebra Linear do Boldrini, na parte sobre
  cadeias de Markov.
 
  --
  Henrique
 
 
 

 =
 Instruções para entrar na lista, sair da lista e usar a lista em
 http://www.mat.puc-rio.br/~obmlistas/obm-l.html
 =



[obm-l] Re: [obm-l] Convergência de Matriz

2011-11-18 Por tôpico Willy George Amaral Petrenko
Esse problema é meio complicado, mas ele é um corolário desse teorema:

http://en.wikipedia.org/wiki/Perron%E2%80%93Frobenius_theorem



2011/11/17 Henrique Rennó henrique.re...@gmail.com

 Suponha a matriz [aij]mxm, onde cada aij representa a probabilidade de um
 evento i levar a um evento j e aij  0 para todo i,j. Calculando as
 potências dessa matriz, é possível provar que ela converge para uma matriz
 P em que todas as colunas são iguais?

 Vi essa propriedade no livro Álgebra Linear do Boldrini, na parte sobre
 cadeias de Markov.

 --
 Henrique




Re: [obm-l] Questao de probabilidade: o sapo e a mosca

2011-10-18 Por tôpico Willy George Amaral Petrenko


 Nossa, isso é lindo! Será que é possível encontrar f(x) em função de
 P(receber x) de modo a garantir vitória com chances maiores que 50%?
 (Interessante que essa estratégia corresponda à intuição de que quanto maior
 for o número, mais sensato é decidir ficar).


 --
 []'s
 Lucas


Na solução eu usei a notação P (receber x) como sendo a probabilidade de
receber o maior número depois que eles já foram sorteados, o que é 50%. E
essa estratégia garante vitória superior a 50%, se f for estritamente
crescente.

Mas eu imagino que vc queira encontrar f em função da distribuição usada
para sortear os números. Bem, se eu sei essa distribuição, basta calcular a
mediana e trocar sempre que meu número for menor que ela. O que no fundo é
escolher f(x)=
0, se x  mediana
1 se x= mediana
Mas não precisa pensar nessa f para chegar a essa conclusão. Trocar se eu
recebi menor que a mediana é bem intuitivo.


Re: [obm-l] Questao de probabilidade: o sapo e a mosca

2011-10-17 Por tôpico Willy George Amaral Petrenko
*Aproveitando o momento probabilistico vejamos tem este problema que estive
pensando e nao consegui:
Voce esta em um programa de auditorio. O apresentador tem dois envelope cada
um com um numero dentro (numeros diferentes).
Voce escolhe um envelope, abre e ve o numero 173.

Ele te pergunta:voce gostaria de trocar o seu envelope por este outro ou
nao?

Ao final se o envelope que voce esolher tiver o maior numero voce ganha
R$1.000.000.000,00 !!!

E ai voce quer trocar ou nao?
Em outras palavras, existe uma estrategia que ele possa adotar para este
jogo que te de uma probabilidade estritamente
maior que 1/2 de vencer?
*

Eu vi certa vez a solução para esse problema e demorei um pouco pra aceitar.

Coisas que vou assumir:
*O programa escolheu dois números aleatórios, regidos por uma distribuição
que eu não conheço, mas que nunca gera 2 números iguais. (ou eu poderia
assumir que se os números forem iguais ganha o jogador)

*Os números já foram determinados e escritos nos envelopes antes de eu
escolher (ou seja não existe essa história de distribuição regida a
questões de meta-jogo, conforme dito pelo Lucas )

*Eu vou escolher um dos envelopes uniformemente, ou seja eu tenho 1/2 de
chance de receber o maior número na minha mão e 1/2 de receber o menor.

* Vou calcular a minha probabilidade antes de abrir o envelope, ou seja, vou
definir minha estratégia, escolher meu envelope e seguir minha estratégia.
Isso é importante porque parece que o enunciado pede pra vc escolher a
estratégia depois de ter o número, o que eu não farei.


Agora eu vou escolher uma função que seja uma função de probabilidade
acumulada estritamente cresente (nada a ver com a função com que foram
escolhidos os números do jogo, que eu não tenho a menor ideia). Ou seja eu
vou entrar no auditório munido de uma função f tal que:

* f seja estritamente crescente, ou seja xy == f(x)  f(y) *[f ser função
de probabilidade acumulada já significa que ela é crescente, mas eu quero
que seja estritamente]*
* f(-infinito) = 0
* f(+infinito) = 1

Por exemplo, f =
(e^x)/2, se x=0
1- (e^-x)/2, se x0

Agora à estratégia: Após receber meu número x, eu fico com o envelope com
probabilidade f(x) e troco com probabilidade 1 - f(x).

Prova de que a estratégia funciona:

Sejam x  y os números nos envelopes (que já foram determinados antes de eu
escolher meu envelope). A minha chance de ganhar seguindo esse estratégia
será:

P(receber x)*P(decidir ficar) + P(receber y)*P(decidir trocar) = 1/2 * f(x)
+ 1/2 * (1 - f(y)) = 1/2 +(f(x) - f(y))/2, e como f é estritamente crescente
e xy, temos que f(x) - f(y) 0 e portanto a minha probabilidade de ganhar é
superior a 1/2!


Considerações:

Essa estratégia tem chance superior a 50% antes de eu escolher meu envelope.
Após eu receber meu número, a minha chance muda para algo que eu não sei.

A minha função pode ser qualquer coisa (dentro do estipulado), e todas elas
funcionarão independente de como foram escolhidos os números. Obviamente
dependendo de qual foi a distribuição escolhida pelo jogo, eu terei f boas e
ruins (que darão probabilidades grandes ou bem próximas de 50%). Por exemplo
se eu sei que os números são inteiros, eu vou querer uma função que só muda
nos inteiros, e permanece constante em [n,n+1). Ou se eu sei que o jogo
sempre escolhe números entre 0 e 1, eu farei uma f tal que f(1) = 1
e f(0) = 0. A função que usei como exemplo só seria boa se os números
estivessem próximos de 0, visto que ela se aproxima dos estremos muito
rapidamente.

Obviamente eu só vou querer usar essa estratégia se eu não sei como foram
escolhidos os números.

Sim, parece mágica, e não, eu não estudei em Hogwarts :)


[obm-l] Re: [obm-l] Re: [obm-l] Sem soluções racionais

2011-09-07 Por tôpico Willy George Amaral Petrenko
Em particular escolha um A tal que x^2+y^2+z^2=7A^2, com mdc (x,y,z,A) =1

2011/9/7 Johann Dirichlet peterdirich...@gmail.com

 Coloca cada cara na forma x =x/A. teremos:

 x^2+y^2+z^2=7A^2, com todas as variáveis inteiras positivas.
 Tentando um módulo 8, acho que sai...

 Em 06/09/11, Vitor Alvesvitor__r...@hotmail.com escreveu:
 
  Não estou conseguinodo resolver o seguinte problema: Prove que não
 existem
  racionais x,y e z tais que x^{2} + y^{2} + z^{2}=7. Qualquer sugestão
 será
  bem vinda,abraços.
 


 --
 /**/
 神が祝福

 Torres

 =
 Instru�ões para entrar na lista, sair da lista e usar a lista em
 http://www.mat.puc-rio.br/~obmlistas/obm-l.html
 =



Re: [obm-l] Desafio limite.

2011-08-29 Por tôpico Willy George Amaral Petrenko
http://en.wikipedia.org/wiki/Knuth's_up-arrow_notation

2011/8/29 Felippe Coulbert Balbi felippeba...@hotmail.com

  Meu amigo Lucas Colucci e eu resolvemos esse problema que surgiu de uma
 aula de calculo.
 Espero que gostem bastante dele.

 Definição: Dado um x pertencendo ao conjunto dos numeros reais e um
 número n pertencendo ao conjunto dos numeros naturais.
 definimos: x|||n= e^(ln(x).x|||n-1)
 definimos: x|||0= 1 (ao invés de x|||n, meu amigo Lucas sugeriu x flecha
 pra cima n, mas enfim, não faz muita diferença)

 Por exemplo:
 x|||3= x^(x^x)
 x|||5= x^(x^(x^(x^x)))

 Prove que

 Lim x|||n =
 x-0+
 =
 1, se n é impar
 0 se n é par

 Grato.
 Coulbert



Re: [obm-l] matriz

2011-08-24 Por tôpico Willy George Amaral Petrenko
0 1 0 0
0 0 1 0
0 0 0 1
0 0 0 0

Para entender a resposta vc deve estar familiarizado com a forma de Jordan:

http://en.wikipedia.org/wiki/Jordan_canonical_form



2011/8/24 Marcus Aurelio Gonçalves Rodrigues marcusaureli...@globo.com

 Alguém pode dar uma ajudinha ai.

 Encontre uma matriz A de dimensão 4 × 4 tal que A, A^2 e A^3 sejam matrizes
 nao nulas, mas A^4 seja a matriz nula.

 --
 Prof Marcus



Re: [obm-l] Quadrado Perfeito

2011-07-28 Por tôpico Willy George Amaral Petrenko
Natália, o menor expoente para o qual a congruência é possível é o número de
carmichael:
http://en.wikipedia.org/wiki/Carmichael_function


Quanto ao problema eu pensei assim:

Se k^2 = 1 + p + p^2 + p^3 + p^4.
Vou estimar o valor de k em função de p.
Parece que k é um pouco maior que p^2 + p/2. De fato (p^2 + p/2)^2 = p^4 +
p^3 + (p^2)/4.
Por outro lado (p^2 + p/2 + 1)^2 = p^4 + p^3 + (9/4)p^2 + p + 1, que é maior
do que a gente gostaria.
Então temos p^2 + p/2  k  p^2 + p/2 +1 == k = p^2 + p/2 + 1/2, visto que
estamos nos inteiros. Daí é só fazer as contas:
k^2 = (p^2 + p/2 + 1/2)^2 = 1 + p + p^2 + p^3 + p^4.
Isso dá uma equação do 2o grau cujas solução são -1 e 3, logo 3 é o único
primo.

Willy


2011/7/28 Nathália Santos nathalia...@hotmail.com

  O phi ao que me referia era o de Euler

 --
 From: joao_maldona...@hotmail.com
 To: obm-l@mat.puc-rio.br
 Subject: RE: [obm-l] Quadrado Perfeito
 Date: Thu, 28 Jul 2011 19:20:44 -0300


  Olá Natália

 Eu acho que está errado a resolução por  4 motivos:


 A= k²= (p^5 -1)/(p-1)
 p^5 -1=k²(p-1)
 p^5 -pk² = 1-k²
 p(p^4 -k²) = 1-k²
 Aplicando congruência módulo p de ambos os lados teremos que:
 1-k² cong 0 (mód p)
 k² cong 1 (mód p)

 A = 1 (mod p) - Na verdade já sabíamos disso não precisava ter feito conta
 nenhuma

 como pelo problema inicial sabemos que p não divide A:
 k^(p-1) cong 1 (mód p) pelo teorema de fermat.
 então p-1 divide 2, 

 Para QUALQUER p  primo diferente de 2, p-1 é par,  também não precisava de
 conta nenhuma

 já que o phi representa o menor expoente para o qual a congruência é
 possível, eu acho rs. 

 Não entendi o phi no problema

 p-1 = 2 ou p-1=1
 p=3 ou p=2
 se p=3 = A=121
 se p=2 = A= 31
 Logo p=3 é a solução.
 Espero que esteja certo, rs. Se não estiver, avise por favor (:
 Espero ter ajudado

 O fato de que p-1 é par implica infinitas soluções na equação. Na verdade
 qualquer  inteiro ímpar,  NÃO SOMENTE O 2 E 3
 Como  k^(p-1)= A^x = 1 (mod p) SEMPRE,  para qualquer p ímpar,
 inclusive 5, 7, 9,   ... 2k+1

 []'s
 João


 --
 From: nathalia...@hotmail.com
 To: obm-l@mat.puc-rio.br
 Subject: RE: [obm-l] Quadrado Perfeito
 Date: Thu, 28 Jul 2011 21:09:19 +

  A= k²= (p^5 -1)/(p-1)
 p^5 -1=k²(p-1)
 p^5 -pk² = 1-k²
 p(p^4 -k²) = 1-k²
 Aplicando congruência módulo p de ambos os lados teremos que:
 1-k² cong 0 (mód p)
 k² cong 1 (mód p)
 como pelo problema inicial sabemos que p não divide A:
 k^(p-1) cong 1 (mód p) pelo teorema de fermat.
 então p-1 divide 2, já que o phi representa o menor expoente para o qual a
 congruência é possível, eu acho rs.
 p-1 = 2 ou p-1=1
 p=3 ou p=2
 se p=3 = A=121
 se p=2 = A= 31
 Logo p=3 é a solução.
 Espero que esteja certo, rs. Se não estiver, avise por favor (:
 Espero ter ajudado
 --
 From: joao_maldona...@hotmail.com
 To: obm-l@mat.puc-rio.br
 Subject: [obm-l] Quadrado Perfeito
 Date: Thu, 28 Jul 2011 17:31:06 -0300

  2000 Grécia:

 Qual o número  primo p, tal que
 A=1 + p + p^2 + p^3 + p^4 é quadrado perfeito?


 A única  coisa que vi é que
 Se p=3  A=121

 Se p não é 3,  e pelo pequeno teorema de fermat um quadrado perfeito deixa
 resto 1 na divisão por 3,  p^4 + p^3 +p^2 + p é divisível por 12,  p^3 + p^2
 + p +1  é divisíivel por 12,  p=6k-1  - (p^2+1)(p+1)

 Acho que não serviu para nada kkk


 []'s
 João




[obm-l] Re: [obm-l] Re: [obm-l] Equação de variáveis inteiras

2011-06-02 Por tôpico Willy George Amaral Petrenko
*03.* Determine todos os pares de inteiros não negativos que são soluções da
equação (xy - 7)^2 = x^2 + y^2.

Vou dar uma solução alternativa para este problema, um pouco mais direta.

x^2*y^2 - 14xy + 49 = x^2 + y^2. == x^2*y^2 - 12xy + 49 = x^2 + +2xy + y^2.
== (xy - 6)^2 + 13 = (x+y)^2.

Agora vc tem um problema do tipo A^2 - B^2 = 13, para A = x+y e B = xy-6. Ou
seja (A-B)(A+B) = 13. Que é fácil resolver. Só tome cuidado que A e B não
são mais positivos, então tem que analisar os 4 casos: (A-B,A+B) =  (1,13) ,
(13,1) , (-1,-13) , (-13,-1).

Abraço


[obm-l] Re: [obm-l] Combinatória e PG

2011-06-02 Por tôpico Willy George Amaral Petrenko
É fácil generalizar isso para inteiros, basta vc considerar um jogo onde
jogam q pessoas, onde q é a razão.
Vou apresentar a generalização para os racionais:

Considere um jogo onde jogam q pessoas e ganham p pessoas (pq) // se vc
teve criatividade para imaginar o jogo acima, não deve ter problemas para
imaginar esse :)

Agora imagine um torneio desse onde jogam q^(n+1) pessoas. Elas jogam
eliminatória, e param de jogar quando sobram p^(n+1) pessoas, que são
consideradas vencedoras.

No round 1 terão q^(n+1) pessoas e q^n jogos;
No round 2 terão q^(n)*p pessoas e q^(n-1)*p jogos;
...
No último round terão q*p^n pessoas e p^n jogos.
Venceram p^(n+1) pessoas após i último round.

Existirão no total q^n + q^(n-1)*p + q^(n-2)*p^2 + ... + q*p^(n-1) + p^n
partidas.

Por outro lado q^(n+1) - p^(n+1) pessoas foram eliminadas (cada pessoa
perdeu apenas 1 vez), e o número de pessoas eliminadas é o número de jogos
vezes o número de pessoas eliminadas por jogo, que é q-p.

Ou seja o número de jogos é (q^(n+1) - p^(n+1)) / (q-p) que é a soma da
PG: q^n + q^(n-1)*p + q^(n-2)*p^2 + ... + q*p^(n-1) + p^n.

Novamente vc tem que multiplicar por algum fator se a_0 for diferente e tem
que tomar cuidado com razão negativas, mas isso é fácil de trabalhar também.

Abraço

2011/5/25 Bernardo Freitas Paulo da Costa bernardo...@gmail.com

 Existem várias formas de se demonstrar a fórmula da soma dos termos de
 uma PG (assim como PA, e outras séries notáveis). Eu estive
 discutindo de tênis (Roland Garros, precisamente) aqui com uns amigos,
 e me veio a seguinte lembrança de uma demonstração muito interessante.

 Problema: Quantos jogos há em Roland Garros? (obs: há 128 jogadores,
 os jogos são eliminatórios, e não há disputa de 3° lugar)
 Resposta: 127, afinal de contas cada jogador, exceto o campeão do
 torneio, perde exatamente um jogo. Mas note que o número de jogos é
 128/2 (primeira rodada) + 128/4 + ... + 1 que é uma soma de PG de
 razão 2. (Ou 1/2, dependendo como você quiser ver. Eu prefiro números
 inteiros, sei lá, parece mais fácil.)  Um campeonato com 2^N jogadores
 mostra que a soma de 2^n (n de 0 a N-1) é 2^N - 1.

 Muito bem. Fazer a soma da PG de razão 2 é fácil. (Não vou falar do
 caso de a_0 != 1, que é óbvio fatorando)

 A idéia agora é a seguinte: arrumar um argumento de contagem (duas
 vezes, para generalizar, como diria a Eureka!) para a soma de PGs de
 razão inteira qualquer. Em seguida, para r real, eu vejo algumas
 pistas: um argumento de abstract nonsense para ver que esse
 resultado válido para todos os inteiros implica no mesmo resultado
 para todos os reais(Álgebra), ou um argumento de aproximação (mas
 tem que mostrar que funciona para todos os racionais; Análise).

 É isso. Divirtam-se.
 --
 Bernardo Freitas Paulo da Costa

 =
 Instruções para entrar na lista, sair da lista e usar a lista em
 http://www.mat.puc-rio.br/~obmlistas/obm-l.html
 =



[obm-l] Re: [obm-l] [obm-l] RE: [obm-l] Número de partições de um conjunto

2011-05-24 Por tôpico Willy George Amaral Petrenko
2011/5/23 Rogerio Ponce abrlw...@gmail.com

 Ola' Paulo e colegas da lista,
 minha sugestao e' calcular de quantas formas podemos dividir as bolas de
 cada cor ( -- #solucoes nao negativas), e multiplicar tudo no final.

 []'s
 Rogerio Ponce


Isso me parece ser a maneira mais simples
Existem 9 maneiras de se dividir 8 bolas identicas entre duas pessoas (e
C(11,3) de dividir entre 4 pessoas). Faz a mesma coisa para as demais e
depois multiplica.
Obtemos 9*11*16, para 2 pessoas e obtemos C(11,3)*C(18,3)*C(13,3) para as 4
pessoas.


[obm-l] Re: [obm-l] RE: [obm-l] Número de partições de um conjunto

2011-05-19 Por tôpico Willy George Amaral Petrenko
Acho que faz sentido ao invés de usar LaTex, usar a imagem, assim fica mais
acessível:
Acho que todo mundo vai conseguir ler (corrijam-me se eu estiver errado).

Bem, me parece que vc quis resolver o problema, não para r e s, mas para
quaisquer 2 conjuntos. A resposta do Paulo está correta para o que pede o
enunciado. Se você quiser calcular para quaisquer 2 conjuntos, tem que tomar
cuidado com o possível termo
pois ele não está sendo contado 2 vezes para vc fazer 1/2*.


[obm-l] Re: [obm-l] Analisar a série usando o critério de comparação

2011-04-24 Por tôpico Willy George Amaral Petrenko
Que tal (2^n)/(n^5)  1, para n suficientemente grande.

2011/4/24 Emanuel Valente emanuelvale...@gmail.com

 Olá pessoal, estou tendo dificuldades em fazer o seguinte exercício:

 Com a ajuda do critério de comparação, analizar a série quanto a
 convergencia e divergencia. Justifique!

 Sum (2^n)/(n^5) , n=1 to n=inf

 obrigado a todos pela atenção desde já

 --
 Emanuel Valente
 Instituto de Física de São Carlos - USP

 =
 Instru�ões para entrar na lista, sair da lista e usar a lista em
 http://www.mat.puc-rio.br/~obmlistas/obm-l.html
 =



Re: [obm-l] quadrado perfeito

2011-04-06 Por tôpico Willy George Amaral Petrenko
Isso é bem fácil mostrar se vc conhece a formula para o numero de divisores
de um numero p1^n1*...*pk^nk que é (n1+1)*...*(nk+1), que pode ser
demonstrada facilmente usando combinatoria


2011/4/6 Samuel Wainer sswai...@hotmail.com

  é verdade que todo numero inteiro quadrado perfeito tem um número impar de
 divisores?

 isso é facil de demonstrar? para os casos mais simples da pra ver que sim.